Đến nội dung

yeutoan2001 nội dung

Có 221 mục bởi yeutoan2001 (Tìm giới hạn từ 02-05-2020)



Sắp theo                Sắp xếp  

#668883 Xét 9 điểm trong mặp phẳng không có 3 điểm nào thẳng hàng

Đã gửi bởi yeutoan2001 on 19-01-2017 - 20:34 trong Tổ hợp và rời rạc

Xét 9 điểm trong mặp phẳng không có 3 điểm nào thẳng hàng 

           Cứ 2 điểm bất kì sẽ được nối với nhau bởi một đường thẳng Hoặc là màu đỏ hoặc xanh

           

a/

        Cứ 3 điểm bất kì thì phải tồn tại ít nhất một đoạn màu đỏ 

  CMR: Nếu Có một điểm nối tối đa là ba đoạn màu xanh thì tồn tại ít nhất một điểm nối với 2 cạnh màu xanh

b/ (Ý kiến của cá nhân)

     

               Với mỗi điểm bắt buộc  nối với các điểm còn lại và chỉ tạo ra 3 đoạn màu xanh

           CM: không thể Thực hiện được 

          (Không biết có chứng minh được 0) 

          

 




#665252 Xin tài liệu thầy VÕ QUỐC BÁ CẨN

Đã gửi bởi yeutoan2001 on 20-12-2016 - 20:02 trong Tài liệu, chuyên đề, phương pháp về Bất đẳng thức

Ở đâu thế bạn  




#665270 Xin tài liệu thầy VÕ QUỐC BÁ CẨN

Đã gửi bởi yeutoan2001 on 20-12-2016 - 21:09 trong Tài liệu, chuyên đề, phương pháp về Bất đẳng thức

Đường dẫn ấy dẫn tới link drive của chính mình nên tụi mình không thể lấy tài liệu của bạn ấy đuwọc chỉ có bạn ấy mới có thể thấy




#665142 Xin tài liệu thầy VÕ QUỐC BÁ CẨN

Đã gửi bởi yeutoan2001 on 19-12-2016 - 17:37 trong Tài liệu, chuyên đề, phương pháp về Bất đẳng thức

Cho Em xxin tài liệu bất đẳng thức của các kì thi chọn đội tuyển của thầy Võ Quốc Bá Cẩn




#675109 x, y, z là các số thực thỏa mãn: $2xyz=3x^{2}+4y^{2}...

Đã gửi bởi yeutoan2001 on 23-03-2017 - 11:54 trong Bất đẳng thức và cực trị

Lời giải hay quá nhưng làm sao để biết được dẫu = xảy ra khi x=y=z, với các bài dấu = xảy ra phức tạp thì phải làm sao???

 Mình chỉ đoán thôi bạn chứ dấu bằng xảy ra xấu thì mình nghĩ phải cân bằng hệ số mà phương pháp ấy khá phức tạp về bước giải phương trình mà không có máy thính thì chịu 




#675086 x, y, z là các số thực thỏa mãn: $2xyz=3x^{2}+4y^{2}...

Đã gửi bởi yeutoan2001 on 23-03-2017 - 00:08 trong Bất đẳng thức và cực trị

Kỹ thuật này dùng phương pháp đồng nhất hệ số. Đầu tiên dùng bất đẳng thức AM-GM hoặc AM-GM suy rộng ta sẽ tìm được giá trị nhỏ nhất khi $x=y=z$ và bài toán quy về chứng minh

\[(3x^2+4y^2+5z^2)(3x+2y+z) - 72xyz \geqslant 0.\]

Vì vế trái là một đa thức bậc $3$ không hoán vị cũng không đối xứng nên nếu biểu diễn được dưới dạng sos thì nó có thể có dạng

\[\begin{aligned}(3x^2+4y^2&+5z^2)(3x+2y+z) - 72xyz =  \\&= (m_1x+m_2y+m_3z)(x-y)^2 + (m_4x+m_5y+m_6z)(y-z)^2 + (m_7x+m_8y+m_9z)(z-x)^2.\end{aligned}\]

Đồng nhất hệ số hai vế giải hệ phương trình tìm nghiệm $m_i \geqslant 0,\,i=1,\,2,\,\ldots,\,9.$ Chú ý rằng hệ phương trình thu được là hệ phương trình tuyến tính và có thể có nhiều nghiệm nên phân tích trên không phải là duy nhất.

 

Có thể xem thêm ở đây.

   Anh có Tool hỗ trợ không ạ chứ em thấy anh biến đổi các bài phức tạp đều về SOS và chỉ cần tiêu chuẩn cơ bản

 

x, y, z là các số thực thỏa mãn: $2xyz=3x^{2}+4y^{2}+5z^{2}$. Tìm giá trị nhỏ nhất: $P=3x+2y+z$

  2xyz=$3x^2+4y^2+5z^2\geq 12\sqrt[12]{x^6y^8z^10}$ 

Suy ra $x^{3}y^{2}z\geq 6^6$

Mà: P=3x+2y+z$\geq 6\sqrt[6]{x^3y^2z}\geq 36$




#666079 Tổng hợp đề thi chuyển hệ học kỳ I lớp 10 CSP

Đã gửi bởi yeutoan2001 on 28-12-2016 - 19:26 trong Thi HSG cấp Tỉnh, Thành phố. Olympic 30-4. Đề thi và kiểm tra đội tuyển các cấp.

Câu:3 Dùng 2 kết quả quen thuộc: 

         $a^{2}+1$ khhông chia hết cho 3 và 4

  Ta cần chỉ ra b chia hết cho cả 3 và 4

            Xét a=3k+1 Dễ dàng => b chia hết cho 3 vì nếu b=3h+1 hoặc 3h+2 thì Vô lí vì $a^{2}+1$ khhông chia hết cho 3

            Xét a=3k+2 Dễ dàng => b chia hết cho 3

Tương tự với TH: CHia hết cho 4




#666097 Tổng hợp đề thi chuyển hệ học kỳ I lớp 10 CSP

Đã gửi bởi yeutoan2001 on 28-12-2016 - 20:39 trong Thi HSG cấp Tỉnh, Thành phố. Olympic 30-4. Đề thi và kiểm tra đội tuyển các cấp.

thôi có lẽ mình hiểu rồi . cảm ơn bạn . 

  Đúng rồi bạn bổ đề đúng với p nguyên tố tuy nhiên một số a=4k+3  thì luôn tồn tại tối thiểu một ước p=4h+3 (p nguyên tố) 




#666078 Tổng hợp đề thi chuyển hệ học kỳ I lớp 10 CSP

Đã gửi bởi yeutoan2001 on 28-12-2016 - 19:15 trong Thi HSG cấp Tỉnh, Thành phố. Olympic 30-4. Đề thi và kiểm tra đội tuyển các cấp.

Câu 2: Giả sử $z< 3$

    Có bất đẳng thức: 

            $\frac{1}{x+yz}+\frac{1}{y+xz}\geq \frac{4}{x+y+xz+yz}> \frac{4}{4x+4y}$(Vô lí)

         => đpcm




#666091 Tổng hợp đề thi chuyển hệ học kỳ I lớp 10 CSP

Đã gửi bởi yeutoan2001 on 28-12-2016 - 20:25 trong Thi HSG cấp Tỉnh, Thành phố. Olympic 30-4. Đề thi và kiểm tra đội tuyển các cấp.

bạn làm tiếp trường hợp chia hết cho 4 đi bạn , đoạn này mới khó . mình cũng chưa nghĩ ra . bạn nên giải chi tiết hơn . đoạn chia hết cho 3 thì mình cũng ra rồi 

  À xin hơi ẩu bài này cần phải dùng thêm bổ đề là $a^{2}+1$ không chia hết cho p: nuyên tố với p=4h+3

Dễ thấy a không thể chẵn vì nếu a chẵn

       a chia hết cho 2 và a+2b chia hết cho2 => $a^{2}+1\vdots 4$ (vô lí)

         Vậy a lẻ a=4k+1 và a=4k+3 (theo bổ đề thì TH này loại vì a chứa ước nguyên tố có dạng 4e+3)

            Vậy bài toán qui về cần chọn b sao cho (4k+1)(4k+1+b)(4k+1+2b) không chia hết cho 4  

                        Chọn b=4h => đpcm

                           Chọn b=4h+1 => 4k+1+2b=4k+1+8h+2=B(4)+3 nên tồn tại p=4l+3 (p nguyên tố) => vô lí theo bổ đề

                            Chọn b=4h+2 => 4k+1+b=4k+1+4h+2=B(4)+3 tương tự trên suy ra vô lí

                            Chọn b=4h+3 => 4k+1+b=4k+1+4h+3=B(4) vô lí vì a^2+1 không chia hết cho 4

 Vậy b chia hết cho 4

  Kí hiệu B(a) là bội của a




#662580 Tìm: $x,y\in \mathbb{N}^*$ thỏa mãn: $3^x=...

Đã gửi bởi yeutoan2001 on 20-11-2016 - 22:47 trong Số học

Xét x lẻ Theo LTE thì v2(3-1)= v2(2)=1 $\geq$ x  Suy ra  x=1; y=1

Xét x chẵn Theo LTE thì v2(3-1)=v2(3-1)+v2(3+1)+v2(x)-1 $\geq$ x 

                                Tương đương 2+v2(x) $\geq$ x

Đặt x=2a.b với (a,b)=1 ;a,b $\geq$ 1

vậy ta có: 2+v2(x) $\geq$ x <=> 2+a $\geq$ 2a.b   Mà 2a.b $\geq$ (1+1)a > a+1  hay 2a.b $\geq$ a+2

    Dấu bằng xảy ra khi b=1; a=2 => x=4 => y=5

Đánh giá trên là BĐT bernoullin nha bạn, không thì chứng minh qui nạp cũng dễ (1+1)a > a+1 với a $\geq$ 3




#661760 Tìm x;y $\epsilon$N* sao cho $2x+1 \vdots y$ và...

Đã gửi bởi yeutoan2001 on 13-11-2016 - 13:26 trong Đại số

Không mất tính tỏng quát giả sử $y\geq x$

CÓ $2x+1\vdots y$ Suy ra $ky=2x+1\leq 2y+1\leq 3y$ Suy ra k=1,2,3 Thay vào vvà dễ dàng tìm ra được 




#666836 Tìm tất cả các số nguyên dương n...

Đã gửi bởi yeutoan2001 on 03-01-2017 - 20:32 trong Số học

Ta dễ có: $n\equiv 0,1,2,3 (mod 4)$ Ta sẽ thu được lần lượt là $2^{n}\equiv 1;2;4;3$ 

      CHẳng hạn xét 1TH: Các TH còn lại tương tự $n\equiv 0 (mod 4) thì

 $n^{2}\equiv 4 (mod 5)$ 

         => $n\equiv 2 (mod5)$

                Dùng định lí phần dư trung hoa để tìm n sẽ có dạng: 

                      $n\equiv 0 (mod 4)$

                      $n\equiv 2 (mod5)$               

                  => $n\equiv 12 (mod20)$

 Các TH còn lại thì tương tự ta sẽ tìm đuwọc các số cần tìm theo môđun 20 




#675533 tìm số $n$

Đã gửi bởi yeutoan2001 on 28-03-2017 - 18:08 trong Số học

Nếu n=1 thì hiển nhiên

Nếu n>1

        p không thể chẵn

        Xét thêm  TH: p=5 =

          Với p khác 5 thì

              Do $a\vdots 5$ và n>1

                 Nên $2^{p}+3^{p}\vdots 5^{2}$

Mà $2^p+3^p=5.(2^{p-1}-2^{p-2}.3+...-2.3^{p-2}+3^{p-1})$

Mà -3 đồng dư với 2 mođulô 5

 Nên $2^{p-1}-2^{p-2}.3+...-2.3^{p-2}+3^{p-1}\equiv p.2^{p-1}(mod5)$

Mà $p.2^{p-1}\not\equiv 0 (mod 5)$ Nên vô lí vì VT không chia hết cho 25




#671906 Tìm Min của $P=(a-1)^{3}+(b-1)^{3}+(c-1)^{3...

Đã gửi bởi yeutoan2001 on 17-02-2017 - 20:25 trong Bất đẳng thức và cực trị

ĐẶT x=a-1  y==b-1  z=c-1  => x,y,z>=-1 Và x+y+z=0

   P=x3+y3+z3 

    Dễ Có (2x-1)2(x+1)>=0 <=> x3>=3/4x-1/4

Tương tự rồi cộng vế theo vế




#663019 Tìm GTLN và GTNN của S=x+y biết $x^{2}+3y^{2}+2xy-10...

Đã gửi bởi yeutoan2001 on 25-11-2016 - 19:16 trong Bất đẳng thức và cực trị

$9=(x+y-5)^{2}+2(y-1)^{2}\geq (x+y+5)^{2} => -2\geq x+y\geq -8$




#662479 Tìm GTLN của $A = \sqrt{1+ x^2} + \sqrt{1 + y^2...

Đã gửi bởi yeutoan2001 on 20-11-2016 - 06:35 trong Bất đẳng thức và cực trị

Dùng Bunhia 

   $(\sqrt{x^{2}+1}.1+\sqrt{2x}.1)^{2}\leqslant (x^{2}+1+2x)(1+1)=2(x+1)^{2}$

   $(\sqrt{x}+\sqrt{y}+\sqrt{z})^{2}\leqslant (x+y+z)(1+1+1)=9$

 

A=$A=\sum (\sqrt{x^{2}+1}+\sqrt{2x})+\sum (3-\sqrt{2})(x+y+z)\leqslant \sqrt{2}(x+1+y+1+z+1)+(3-\sqrt{2})(3)$




#664113 Tìm bộ số thỏa mãn

Đã gửi bởi yeutoan2001 on 07-12-2016 - 22:39 trong Số học

đoạn màu đỏ vì sao lại được như vậy bạn 

Có $(m+n)^{3}-8\vdots m^{2}+n^{2} <=> (m+n)(m^{2}+n^{2}+2mn-2m-2n+4)\vdots m^{2}+n^{2}$

 DO $p=m^{2}+n^{2}$ là số nguyên tố đó  




#664110 Tìm bộ số thỏa mãn

Đã gửi bởi yeutoan2001 on 07-12-2016 - 22:19 trong Số học

$m^{3}+n^{3}-4\vdots m^{2}+n^{2} <=> (m+n)mn+4\vdots m^{2}+n^{2}$

$m^{3}+n^{3}-4+3mn(m+n)+12\vdots m^{2}+n^{2} => (m+n)^{3}-8\vdots m^{2}+n^{2}$

 => 2TH: 

            TH1 $m+n\vdots m^{2}+n^{2}$ Suy ra m=n=1 vì nếu m,n>1 ta dễ chứng minh $m^{2}+n^{2}$$>=m+n

            TH2: $2mn-2m-2n+4\vdots m^{2}+n^{2} => 2mn-2m-2n+4\geq m^{2}+n^{2} => 4+2mn>2mn-2m-2n+4>m^{2}+n^{2} => 4>(m-n)^{2}$

                       (dễ dàng tìm được m,n) 

Tìm bộ các số nguyên dương $(m;n)$ sao cho $p=m^2+n^2$ là số nguyên tố và $m^3+n^3-4$ chia hết cho $p$

 



#687745 Tuyển tập các chuyên đề Hình học trên Tạp chí THTT

Đã gửi bởi yeutoan2001 on 16-07-2017 - 21:19 trong Tài liệu, chuyên đề, phương pháp về Hình học

File tổng hợp các chuyên đề hình học trong các số của TC THTT 

   Bị sao rồi ấy tải không được 
          bạn sửa dùm Cảm Ơn




#660508 Topic về tổ hợp, các bài toán về tổ hợp

Đã gửi bởi yeutoan2001 on 03-11-2016 - 21:09 trong Tổ hợp và rời rạc

Có 18 người chứng minh rằng cs 4 người đôi một quen nhau hoặc đôi một 0 quen nhau




#663212 Topic : Bất đẳng thức chứa biến ở mũ

Đã gửi bởi yeutoan2001 on 27-11-2016 - 18:24 trong Bất đẳng thức - Cực trị

Các bạn cho mình hỏi Bất đẳng thức Bernoulin chỉ dùng cho mũ nguyên thôi đúng không




#663209 Topic : Bất đẳng thức chứa biến ở mũ

Đã gửi bởi yeutoan2001 on 27-11-2016 - 18:21 trong Bất đẳng thức - Cực trị

Sử dụng $AM-GM$ suy rộng, ta có :
$$1.a^bb^cc^a \le \left (\dfrac{ab + bc + ca}{a + b + c}\right )^{a + b + c} = ab + bc + ca \le \dfrac{(ab + bc + ca)^2}{3} = \dfrac{1}{3}$$
$$2.a^{3c + b}b^{3a + c}c^{3b + a} \le \left (\dfrac{a(3c + b) + b(3a + c) + c(3b + a)}{4(a + b + c)}\right )^{4(a + b + c)}$$ $$ = \left (ab + bc + ca \right )^4 \le \dfrac{1}{3^4} = \dfrac{1}{81}$$

Bạn ơi a,b,c không nguyên sao cô si cho a+b+c số được 




#663700 Phương trình nghiệm nguyên dương dạng: $(x+1)^y-x^z=1$

Đã gửi bởi yeutoan2001 on 03-12-2016 - 15:11 trong Số học

 

Xét phương trình $(x+1)^y=x^z+1$ 
Dễ thấy $z$ lẻ 
PT $\Leftrightarrow (x+1)^{y-1}=x^{z-1}-x^{z-2}+..+x^2-x+1$ (*)

  • Nếu $x$ là số lẻ khi đó mỗi số hạng của vế phải  của (*) đều lẻ nên suy ra vế phải của (*) là lẻ. Mặc khác vế trái lại chẵn (với $y>1$ còn khi $y=1$ thì $x=1$ $z=t$ với $t$ nguyên dương) dẫn đến vô lí (ý em là $y>1$) 
  • Nếu $x$ là số chẵn viết lại phương trình $x^z=(x+1)^y-1$ (1)
    Suy ra $x^z=C_y^1.x+C_y^2.x^2+...+C_y^y.x^y$ 
    Dễ thấy $z>1$ suy ra $x^{z-1}=C_y^1+C_y^2.x+...+C_y^y.x^{y-1}$ (2)
    Xét $y=z=1$ suy ra $x=t$ với $t$ nguyên dương 
    Với $y,z>1$ vậy thì $x|x^{z-1},x^{y-1}$. Từ (2) suy ra $x|y$ . Vì $x$ chẵn nên $y$ chẵn 
    Đặt $x=2a,y=2b$ trong đó $a,b \ge 1$  
    (1) $\Rightarrow (2a)^z=(2a+1)^{2b}-1 \Rightarrow (2a)^z=[(2a+1)^b-1][(2a+1)^b+1]$  (3)
    Đặt $d=gcd((2a+1)^b-1,(2a+1)^b+1) \Rightarrow d=2$ (4)
    Nên ta tiếp tục chia thành các trường hợp : 
    TH1 : $a=1$ hoặc $a$ là lũy thừa của $2$ suy ra $(2a)^z$ cũng là lũy thừa $2$  
    (3)+(4) $\rightarrow (2a+1)^y-1=2 \Leftrightarrow a=1,b=1 \Rightarrow x=y=2,z=3$ 
    TH2 : $a$ không là lũy thừa của $2$ suy ra $a^z>2^z$ 
    Ta có $2^z.a^z=[(2a+1)^b-1][(2a+1)^b+1]$ do $a$ không là lũy thừa của $2$ nên $a^z$ là số lẻ. 
    Từ đó suy ra $\begin{cases} &(2b+1)^y+1=2^{z_1}.a^z&\\&(2b+1)^y-1=2^{z_2}& \end{cases}$ ($z_1+z_2=z$ và $z_1,z_2 \in \mathbb{N^*}$) 
    $a^z>2^z \Rightarrow 2^{z_1}.a^z>2^{z_1+z}$ 
    Suy ra $2^{z_1}.a^z-2^{z_2}>2$ 
    Suy ra $[(2a+1)^b+1]-[(2a+1)^b-1]>2$ (dễ thấy vô lí) 
    Kết luận $(x,y,z)=(1,1,t),(t,1,1),(2,2,3)$ 
    P/s : Xem lại dùm em cái

 

Bạn ơi chỗ đó a đâu có nguyên tố đâu nhỡ a được phân tích thành nhân tử có dạng a=pq thì 

$(2a+1)^{b}-1=2.p^{z} VÀ (2a+1)^{b}+1=2^{z-1}.q^{z}$ cũng được mà Mình nghĩ a phải nguyên tố bạn mới được tách như thế




#663874 Phương trình nghiệm nguyên dương dạng: $(x+1)^y-x^z=1$

Đã gửi bởi yeutoan2001 on 05-12-2016 - 19:31 trong Số học

 

Xét phương trình $(x+1)^y=x^z+1$ 
Dễ thấy $z$ lẻ 
PT $\Leftrightarrow (x+1)^{y-1}=x^{z-1}-x^{z-2}+..+x^2-x+1$ (*)

  • Nếu $x$ là số lẻ khi đó mỗi số hạng của vế phải  của (*) đều lẻ nên suy ra vế phải của (*) là lẻ. Mặc khác vế trái lại chẵn (với $y>1$ còn khi $y=1$ thì $x=1$ $z=t$ với $t$ nguyên dương) dẫn đến vô lí (ý em là $y>1$) 
  • Nếu $x$ là số chẵn viết lại phương trình $x^z=(x+1)^y-1$ (1)
    Suy ra $x^z=C_y^1.x+C_y^2.x^2+...+C_y^y.x^y$ 
    Dễ thấy $z>1$ suy ra $x^{z-1}=C_y^1+C_y^2.x+...+C_y^y.x^{y-1}$ (2)
    Xét $y=z=1$ suy ra $x=t$ với $t$ nguyên dương 
    Với $y,z>1$ vậy thì $x|x^{z-1},x^{y-1}$. Từ (2) suy ra $x|y$ . Vì $x$ chẵn nên $y$ chẵn 
    Đặt $x=2a,y=2b$ trong đó $a,b \ge 1$  
    (1) $\Rightarrow (2a)^z=(2a+1)^{2b}-1 \Rightarrow (2a)^z=[(2a+1)^b-1][(2a+1)^b+1]$  (3)
    Đặt $d=gcd((2a+1)^b-1,(2a+1)^b+1) \Rightarrow d=2$ (4)
    Nên ta tiếp tục chia thành các trường hợp : 
    TH1 : $a=1$ hoặc $a$ là lũy thừa của $2$ suy ra $(2a)^z$ cũng là lũy thừa $2$  
    (3)+(4) $\rightarrow (2a+1)^y-1=2 \Leftrightarrow a=1,b=1 \Rightarrow x=y=2,z=3$ 
    TH2 : $a$ không là lũy thừa của $2$ suy ra $a^z>2^z$ 
    Ta có $2^z.a^z=[(2a+1)^b-1][(2a+1)^b+1]$ do $a$ không là lũy thừa của $2$ nên $a^z$ là số lẻ. 
    Từ đó suy ra $\begin{cases} &(2b+1)^y+1=2^{z_1}.a^z&\\&(2b+1)^y-1=2^{z_2}& \end{cases}$ ($z_1+z_2=z$ và $z_1,z_2 \in \mathbb{N^*}$) 
    $a^z>2^z \Rightarrow 2^{z_1}.a^z>2^{z_1+z}$ 
    Suy ra $2^{z_1}.a^z-2^{z_2}>2$ 
    Suy ra $[(2a+1)^b+1]-[(2a+1)^b-1]>2$ (dễ thấy vô lí) 
    Kết luận $(x,y,z)=(1,1,t),(t,1,1),(2,2,3)$ 
    P/s : Xem lại dùm em cái

 

Không phải bạn ơi số a sao chia ra đuwọc 2 TH: LÀ lũy thừa của a và Trường hợp lẻ được còn một trường hợp là $a=2^{m}.n$